Skip to main content
Erschienen in: Journal of Inequalities and Applications 1/2015

Open Access 01.12.2015 | Research

On the partial finite sums of the reciprocals of the Fibonacci numbers

verfasst von: Andrew YZ Wang, Peibo Wen

Erschienen in: Journal of Inequalities and Applications | Ausgabe 1/2015

Aktivieren Sie unsere intelligente Suche, um passende Fachinhalte oder Patente zu finden.

search-config
loading …

Abstract

In this article, we obtain two interesting families of partial finite sums of the reciprocals of the Fibonacci numbers, which substantially improve two recent results involving the reciprocal Fibonacci numbers. In addition, we present an alternative and elementary proof of a result of Wu and Wang.
Hinweise

Competing interests

The authors declare that they have no competing interests.

Authors’ contributions

All authors contributed equally to deriving all the results of this article, and read and approved the final manuscript.

1 Introduction

The Fibonacci sequence [1], Sequence A000045 is defined by the linear recurrence relation
$$F_{n}=F_{n-1}+F_{n-2}\quad \mbox{for }n\geq2, $$
where \(F_{n}\) is the nth Fibonacci number with \(F_{0}=0\) and \(F_{1}=1\). There exists a simple and non-obvious formula for the Fibonacci numbers,
$$F_{n}=\frac{1}{\sqrt{5}} \biggl(\frac{1+\sqrt{5}}{2} \biggr)^{n}- \frac{1}{\sqrt{5}} \biggl(\frac{1-\sqrt{5}}{2} \biggr)^{n}. $$
The Fibonacci sequence plays an important role in the theory and applications of mathematics, and its various properties have been investigated by many authors; see [25].
In recent years, there has been an increasing interest in studying the reciprocal sums of the Fibonacci numbers. For example, Elsner et al. [69] investigated the algebraic relations for reciprocal sums of the Fibonacci numbers. In [10], the partial infinite sums of the reciprocal Fibonacci numbers were studied by Ohtsuka and Nakamura. They established the following results, where \(\lfloor\cdot\rfloor\) denotes the floor function.
Theorem 1.1
For all \(n\geq2\),
$$ \Biggl\lfloor \Biggl(\sum_{k=n}^{\infty} \frac{1}{F_{k}} \Biggr)^{-1} \Biggr\rfloor = \left \{ \begin{array}{l@{\quad}l} F_{n-2}, &\textit{if }n\textit{ is even}; \\ F_{n-2}-1, &\textit{if }n\textit{ is odd}. \end{array} \right . $$
(1.1)
Theorem 1.2
For each \(n\geq1\),
$$ \Biggl\lfloor \Biggl(\sum_{k=n}^{\infty} \frac{1}{F_{k}^{2}} \Biggr)^{-1} \Biggr\rfloor = \left \{ \begin{array}{l@{\quad}l} F_{n}F_{n-1}-1, &\textit{if }n\textit{ is even}; \\ F_{n}F_{n-1}, &\textit{if }n\textit{ is odd}. \end{array} \right . $$
(1.2)
Further, Wu and Zhang [11, 12] generalized these identities to the Fibonacci polynomials and Lucas polynomials and various properties of such polynomials were obtained.
Recently, Holliday and Komatsu [13] considered the generalized Fibonacci numbers which are defined by
$$G_{n+2}=aG_{n+1}+G_{n},\quad n\geq0 $$
with \(G_{0}=0\) and \(G_{1}=1\), and a is a positive integer. They showed that
$$ \Biggl\lfloor \Biggl(\sum_{k=n}^{\infty} \frac{1}{G_{k}} \Biggr)^{-1} \Biggr\rfloor = \left \{ \begin{array}{l@{\quad}l} G_{n}-G_{n-1}, &\mbox{if }n\mbox{ is even and }n\geq2; \\ G_{n}-G_{n-1}-1, &\mbox{if }n\mbox{ is odd and }n\geq1 \end{array} \right . $$
(1.3)
and
$$ \Biggl\lfloor \Biggl(\sum_{k=n}^{\infty} \frac{1}{G_{k}^{2}} \Biggr)^{-1} \Biggr\rfloor = \left \{ \begin{array}{l@{\quad}l} aG_{n}G_{n-1}-1, &\mbox{if }n\mbox{ is even and }n\geq2; \\ aG_{n}G_{n-1}, &\mbox{if }n\mbox{ is odd and }n\geq1. \end{array} \right . $$
(1.4)
More recently, Wu and Wang [14] studied the partial finite sum of the reciprocal Fibonacci numbers and deduced that, for all \(n\geq4\),
$$ \Biggl\lfloor \Biggl(\sum_{k=n}^{2n} \frac{1}{F_{k}} \Biggr)^{-1} \Biggr\rfloor =F_{n-2}. $$
(1.5)
Inspired by Wu and Wang’s work, we obtain two families of partial finite sums of the reciprocal Fibonacci numbers in this paper, which significantly improve Ohtsuka and Nakamura’s results, Theorems 1.1 and 1.2. In addition, we present an alternative proof of (1.5).

2 Reciprocal sum of the Fibonacci numbers

We first present several well known results on Fibonacci numbers, which will be used throughout the article. The detailed proofs can be found in [5].
Lemma 2.1
Let \(n\geq1\), we have
$$ F_{n}^{2}-F_{n-1}F_{n+1}=(-1)^{n-1} $$
(2.1)
and
$$ F_{a}F_{b}+F_{a+1}F_{b+1}=F_{a+b+1} $$
(2.2)
if a and b are positive integers.
As a consequence of (2.2), we have the following result.
Corollary 2.2
For all \(n\geq1\), we have
$$\begin{aligned}& F_{2n} = F_{n-1}F_{n}+F_{n}F_{n+1}, \end{aligned}$$
(2.3)
$$\begin{aligned}& F_{2n+1} = F_{n}^{2}+F_{n+1}^{2}, \end{aligned}$$
(2.4)
$$\begin{aligned}& F_{2n+1} = F_{n-1}F_{n+1}+F_{n}F_{n+2}. \end{aligned}$$
(2.5)
It is easy to derive the following lemma and we leave the proof as a simple exercise.
Lemma 2.3
For each \(n\geq1\), we have
$$ F_{n+1}F_{n+2}-F_{n-1}F_{n}=F_{2n+1}. $$
(2.6)
We now establish two inequalities on Fibonacci numbers which will be used later.
Lemma 2.4
If \(n\geq6\), then
$$ F_{n-2}F_{n-1}>F_{n+1}. $$
(2.7)
Proof
It is easy to see that
$$\begin{aligned} F_{n-2}F_{n-1}-F_{n+1} =&F_{n-2}F_{n-1}-(F_{n-1}+F_{n}) \\ =&F_{n-2}F_{n-1}-F_{n-1}-(F_{n-2}+F_{n-1}) \\ =&(F_{n-2}-2)F_{n-1}-F_{n-2}. \end{aligned}$$
Since \(n\geq6\), \(F_{n-2}-2>1\). So
$$F_{n-2}F_{n-1}-F_{n+1}>F_{n-1}-F_{n-2}>0, $$
which completes the proof. □
Lemma 2.5
For each \(n\geq3\), we have
$$ F_{3n-1}(F_{n}+F_{n-3})>F_{n-2}F_{n-1}F_{n}F_{n+1}. $$
(2.8)
Proof
Applying (2.2), we get
$$F_{3n-1}=F_{n-1}F_{2n-1}+F_{n}F_{2n}. $$
Thus
$$F_{3n-1}(F_{n}+F_{n-3})\geq(F_{n-1}F_{2n-1}+F_{n}F_{2n})F_{n}>F_{n}^{2}F_{2n}> F_{n-1}F_{n}F_{2n}. $$
Employing (2.3), we have
$$F_{2n}>F_{n}F_{n+1}>F_{n-2}F_{n+1}, $$
which yields the desired equation (2.8). □
The following are some inequalities on the sum of reciprocal Fibonacci numbers.
Proposition 2.6
For all \(n\geq2\), we have
$$ \sum_{k=n}^{2n} \frac{1}{F_{k}}> \frac{1}{F_{n-2}+1}. $$
(2.9)
Proof
For all \(k\geq2\),
$$\begin{aligned} \frac{1}{F_{k-2}+1}-\frac{1}{F_{k}}-\frac{1}{F_{k-1}+1} =&\frac {F_{k}-F_{k-2}-1}{F_{k}(F_{k-2}+1)} - \frac{1}{F_{k-1}+1} \\ =&\frac {(F_{k-1}-1)(F_{k-1}+1)-F_{k}F_{k-2}-F_{k}}{(F_{k-2}+1)(F_{k-1}+1)F_{k}} \\ =&\frac{F_{k-1}^{2}-F_{k}F_{k-2}-1-F_{k}}{(F_{k-2}+1)(F_{k-1}+1)F_{k}}. \end{aligned}$$
Invoking (2.1), we obtain \(F_{k-1}^{2}-F_{k}F_{k-2}=(-1)^{k}\). Therefore,
$$ \frac{1}{F_{k-2}+1}-\frac{1}{F_{k}}-\frac{1}{F_{k-1}+1}= \frac{(-1)^{k}-1-F_{k}}{(F_{k-2}+1)(F_{k-1}+1)F_{k}}. $$
Now we have
$$\begin{aligned} \sum_{k=n}^{2n}\frac{1}{F_{k}} =& \frac{1}{F_{n-2}+1}-\frac{1}{F_{2n-1}+1}+ \sum_{k=n}^{2n} \frac{(-1)^{k-1}+1+F_{k}}{(F_{k-2}+1)(F_{k-1}+1)F_{k}} \\ >&\frac{1}{F_{n-2}+1}-\frac{1}{F_{2n-1}+1}+ \sum_{k=n}^{2n} \frac{1}{(F_{k-2}+1)(F_{k-1}+1)} \\ >&\frac{1}{F_{n-2}+1}+\frac{1}{(F_{n-2}+1)(F_{n-1}+1)}-\frac{1}{F_{2n-1}+1}. \end{aligned}$$
Because of (2.5), we have
$$\begin{aligned} F_{2n-1}+1-(F_{n-2}+1) (F_{n-1}+1) =&F_{2n-1}-F_{n-2}F_{n-1}-F_{n-2}-F_{n-1} \\ =&F_{n-2}^{2}+F_{n-1}F_{n+1}-F_{n} \\ >&0. \end{aligned}$$
Thus, we arrive at
$$ \sum_{k=n}^{2n}\frac{1}{F_{k}}> \frac{1}{F_{n-2}+1}. $$
This completes the proof. □
Proposition 2.7
Assume that \(m\geq2\). Then, for all even integers \(n\geq4\), we have
$$ \sum_{k=n}^{mn} \frac{1}{F_{k}}< \frac{1}{F_{n-2}}. $$
(2.10)
Proof
By elementary manipulations and (2.1), we deduce that
$$ \frac{1}{F_{k-2}}-\frac{1}{F_{k}}-\frac{1}{F_{k-1}}=\frac {(-1)^{k}}{F_{k-2}F_{k-1}F_{k}},\quad k\geq3. $$
Hence, for \(n\geq3\), we have
$$ \sum_{k=n}^{mn} \frac{1}{F_{k}}=\frac{1}{F_{n-2}}-\frac{1}{F_{mn-1}}+ \sum _{k=n}^{mn}\frac{(-1)^{k-1}}{F_{k-2}F_{k-1}F_{k}}. $$
(2.11)
Since n is even,
$$\sum_{k=n}^{mn}\frac{(-1)^{k-1}}{F_{k-2}F_{k-1}F_{k}}< 0, $$
from which we conclude that
$$ \sum_{k=n}^{mn}\frac{1}{F_{k}}< \frac{1}{F_{n-2}}. $$
The proof is complete. □
Proposition 2.8
If \(n\geq5\) is odd, then
$$ \sum_{k=n}^{2n} \frac{1}{F_{k}}< \frac{1}{F_{n-2}}. $$
(2.12)
Proof
It is straightforward to check that the statement is true when \(n=5\).
Now we assume that \(n\geq7\). Since n is odd, we have
$$\sum_{k=n+1}^{2n}\frac{(-1)^{k-1}}{F_{k-2}F_{k-1}F_{k}}< 0. $$
Applying (2.7) and (2.6) yields
$$\begin{aligned} \frac{1}{F_{n-2}F_{n-1}F_{n}}-\frac{1}{F_{2n-1}} < &\frac{1}{F_{n}F_{n+1}}-\frac{1}{F_{2n-1}} \\ =&\frac{F_{2n-1}-F_{n}F_{n+1}}{F_{n}F_{n+1}F_{2n-1}} \\ =&-\frac{F_{n-2}F_{n-1}}{F_{n}F_{n+1}F_{2n-1}} \\ <&0. \end{aligned}$$
Employing (2.11) and the above two inequalities, (2.12) follows immediately. □
Proposition 2.9
Let \(m\geq3\) be given. If \(n\geq3\) is odd, we have
$$ \sum_{k=n}^{mn} \frac{1}{F_{k}}>\frac{1}{F_{n-2}}. $$
(2.13)
Proof
It is easy to see that
$$\sum_{k=3}^{3m}\frac{1}{F_{k}}> \frac{1}{ F_{1}}, $$
thus (2.13) holds for \(n=3\). Now we assume that \(n\geq 5\).
Based on (2.11) and using the fact n is odd, we have
$$\sum_{k=n}^{mn}\frac{1}{F_{k}}= \frac{1}{F_{n-2}}+\frac{1}{F_{n-2}F_{n-1}F_{n}}- \frac{1}{F_{n-1}F_{n}F_{n+1}}-\frac{1}{F_{mn-1}}+ \sum _{k=n+2}^{mn}\frac{(-1)^{k-1}}{F_{k-2}F_{k-1}F_{k}}. $$
It is clear that
$$\sum_{k=n+2}^{mn}\frac{(-1)^{k-1}}{F_{k-2}F_{k-1}F_{k}}>0. $$
Since \(m\geq3\) and invoking (2.8), we obtain
$$F_{mn-1}(F_{n-3}+F_{n})\geq F_{3n-1}(F_{n-3}+F_{n})>F_{n-2}F_{n-1}F_{n}F_{n+1}, $$
which implies
$$\begin{aligned} \frac{1}{F_{n-2}F_{n-1}F_{n}}- \frac{1}{F_{n-1}F_{n}F_{n+1}}-\frac{1}{F_{mn-1}} =& \frac{F_{n+1}-F_{n-2}}{F_{n-2}F_{n-1}F_{n}F_{n+1}}- \frac{1}{F_{mn-1}} \\ =&\frac{F_{n-3}+F_{n}}{F_{n-2}F_{n-1}F_{n}F_{n+1}}- \frac{F_{n-3}+F_{n}}{F_{mn-1}(F_{n-3}+F_{n})} \\ >&0. \end{aligned}$$
Therefore, (2.13) also holds for \(n\geq5\). □
Now we state our main results on the sum of reciprocal Fibonacci numbers.
Theorem 2.10
For all \(n\geq4\), we have
$$ \Biggl\lfloor \Biggl(\sum_{k=n}^{2n} \frac{1}{F_{k}} \Biggr)^{-1} \Biggr\rfloor =F_{n-2}. $$
(2.14)
Proof
Combining (2.9), (2.10), and (2.12), we conclude that, for all \(n\geq4\),
$$\frac{1}{F_{n-2}+1}< \sum_{k=n}^{2n} \frac{1}{F_{k}}<\frac{1}{F_{n-2}}, $$
from which (2.14) follows immediately. □
Remark
Identity (2.14) was first conjectured by Professor Ohtsuka, the first author of [10]. Based on the formula of \(F_{n}\) and using analytic methods, Wu and Wang [14] presented a proof of (2.14). In contrast to Wu and Wang’s work, the techniques we use here are more elementary.
Theorem 2.11
If \(m\geq3\) and \(n\geq2\), then
$$ \Biggl\lfloor \Biggl(\sum_{k=n}^{mn} \frac{1}{F_{k}} \Biggr)^{-1} \Biggr\rfloor = \left \{ \begin{array}{l@{\quad}l} F_{n-2}, &\textit{if }n\textit{ is even}; \\ F_{n-2}-1, &\textit{if }n\textit{ is odd}. \end{array} \right . $$
(2.15)
Proof
It is clear that
$$ \Biggl\lfloor \Biggl(\sum_{k=2}^{2m} \frac{1}{F_{k}} \Biggr)^{-1} \Biggr\rfloor =F_{0}. $$
(2.16)
Combining (2.9) and (2.10), we find that, for all even integers \(n\geq4\),
$$ \frac{1}{F_{n-2}+1}< \sum_{k=n}^{mn} \frac{1}{F_{k}}<\frac{1}{F_{n-2}}. $$
(2.17)
Thus (2.16) and (2.17) show that, for all \(m\geq3\),
$$\Biggl\lfloor \Biggl(\sum_{k=n}^{mn} \frac{1}{F_{k}} \Biggr)^{-1} \Biggr\rfloor =F_{n-2}, $$
provided that \(n\geq2\) is even.
Next we aim to prove that, for \(m\geq3\) and all odd integers \(n\geq 3\),
$$ \Biggl\lfloor \Biggl(\sum_{k=n}^{mn} \frac{1}{F_{k}} \Biggr)^{-1} \Biggr\rfloor =F_{n-2}-1. $$
(2.18)
If \(n=3\), we can readily see that
$$\sum_{k=3}^{3m}\frac{1}{F_{k}}>1, $$
thus (2.18) holds for \(n=3\). So in the rest of the proof we assume that \(n\geq5\).
It is not hard to derive that, for all \(k\geq5\),
$$\frac{1}{F_{k-2}-1}-\frac{1}{F_{k}}-\frac{1}{F_{k-1}-1}= \frac{(-1)^{k}-1+F_{k}}{F_{k}(F_{k-2}-1)(F_{k-1}-1)}>0. $$
Hence, we get
$$ \sum_{k=n}^{mn} \frac{1}{F_{k}}< \frac{1}{F_{n-2}-1}-\frac{1}{F_{mn-1}-1} <\frac{1}{F_{n-2}-1}. $$
(2.19)
Finally, combining (2.19) with (2.13) yields (2.18). □
Remark
As \(m\rightarrow\infty\), (2.15) becomes (1.1). Hence our result, Theorem 2.11, substantially improves Theorem 1.1.

3 Reciprocal square sum of the Fibonacci numbers

We first give several preliminary results which will be used in our later proofs.
Lemma 3.1
For all \(n\geq1\),
$$ F_{n}F_{n+1}-F_{n-1}F_{n+2}=(-1)^{n-1}. $$
(3.1)
Proof
It is easy to show that
$$\begin{aligned} F_{n}F_{n+1}-F_{n-1}F_{n+2} =&F_{n}F_{n+1}-F_{n-1}(F_{n}+F_{n+1}) \\ =&F_{n}F_{n+1}-F_{n}F_{n-1}-F_{n-1}F_{n+1} \\ =&F_{n}(F_{n+1}-F_{n-1})-F_{n-1}F_{n+1} \\ =&F_{n}^{2}-F_{n-1}F_{n+1}. \end{aligned}$$
Employing (2.1), the desired result follows. □
Proposition 3.2
Given an integer \(m\geq2\) and let \(n\geq3\) be odd, we have
$$ \sum_{k=n}^{mn} \frac{1}{F_{k}^{2}}< \frac{1}{F_{n-1}F_{n}}. $$
(3.2)
Proof
It is straightforward to check that, for each \(k\geq2\),
$$\begin{aligned}& \frac{1}{F_{k-1}F_{k}}-\frac{1}{F_{k}^{2}} - \frac{1}{F_{k+1}^{2}}-\frac {1}{F_{k+1}F_{k+2}} \\& \quad = \frac {F_{k}F_{k+1}^{2}F_{k+2}-F_{k-1}F_{k+1}^{2}F_{k+2}-F_{k-1}F_{k}^{2}F_{k+2}-F_{k-1}F_{k}^{2} F_{k+1}}{F_{k-1}F_{k}^{2}F_{k+1}^{2}F_{k+2}} \\& \quad = \frac{F_{k}F_{k+1}(F_{k+1}F_{k+2}-F_{k-1}F_{k})-F_{k-1}F_{k+2}(F_{k+1}^{2}+F_{k}^{2})}{ F_{k-1}F_{k}^{2}F_{k+1}^{2}F_{k+2}} \\& \quad = \frac{(F_{k}F_{k+1}-F_{k-1}F_{k+2})F_{2k+1}}{ F_{k-1}F_{k}^{2}F_{k+1}^{2}F_{k+2}} \\& \quad = \frac{(-1)^{k-1}F_{2k+1}}{F_{k-1}F_{k}^{2}F_{k+1}^{2}F_{k+2}}, \end{aligned}$$
where the last equality follows from (3.1).
Since n is odd, we have
$$\frac{1}{F_{n-1}F_{n}}-\frac{1}{F_{n}^{2}}-\frac{1}{F_{n+1}^{2}}-\frac {1}{F_{n+1}F_{n+2}}>0. $$
If m is even, then
$$\sum_{k=n}^{mn}\frac{1}{F_{k}^{2}}< \frac{1}{F_{n-1}F_{n}}-\frac{1}{F_{mn}F_{mn+1}} <\frac{1}{F_{n-1}F_{n}}. $$
If m is odd, then
$$\sum_{k=n}^{mn}\frac{1}{F_{k}^{2}}< \frac{1}{F_{n-1}F_{n}}-\frac{1}{F_{mn-1}F_{mn}}+ \frac{1}{F_{mn}^{2}}<\frac{1}{F_{n-1}F_{n}}. $$
Thus, (3.2) always holds. □
Proposition 3.3
Let n be odd, then we have
$$ \sum_{k=n}^{2n} \frac{1}{F_{k}^{2}}> \frac{1}{F_{n-1}F_{n}+1}. $$
(3.3)
Proof
Invoking (2.1), we can readily derive that
$$ \frac{1}{F_{k-1}F_{k}+1}-\frac{1}{F_{k}^{2}}-\frac{1}{F_{k}F_{k+1}+1}=\left \{ \begin{array}{l@{\quad}l} -\frac{2F_{k-1}F_{k}+1}{F_{k}^{2}(F_{k-1}F_{k}+1)(F_{k}F_{k+1}+1)}, &\mbox{if }k \mbox{ is odd}; \\ -\frac{2F_{k}F_{k+1}+1}{F_{k}^{2}(F_{k-1}F_{k}+1)(F_{k}F_{k+1}+1)}, &\mbox{if } k\mbox{ is even}. \end{array} \right . $$
Now we have
$$\begin{aligned} \sum_{k=n}^{2n}\frac{1}{F_{k}^{2}} =& \frac{1}{F_{n-1}F_{n}+1}+\biggl( \frac{2F_{n-1}F_{n}+1}{F_{n}^{2}(F_{n-1}F_{n}+1)(F_{n}F_{n+1}+1)} \\ &{}+\frac {2F_{n+1}F_{n+2}+1}{F_{n+1}^{2}(F_{n}F_{n+1}+1)(F_{n+1}F_{n+2}+1)}+\cdots \\ &{}+\frac {2F_{2n}F_{2n+1}+1}{F_{2n}^{2}(F_{2n-1}F_{2n}+1)(F_{2n}F_{2n+1}+1)}\biggr)- \frac{1}{F_{2n}F_{2n+1}+1} \\ >&\frac{1}{F_{n-1}F_{n}+1}+\frac {2F_{n-1}F_{n}+1}{F_{n}^{2}(F_{n-1}F_{n}+1)(F_{n}F_{n+1}+1)}- \frac{1}{F_{2n}F_{2n+1}+1}. \end{aligned}$$
It is obvious that \(2F_{k-1}F_{k}\geq F_{k-1}F_{k+1}\). From (2.1) and the fact that n is odd, we obtain
$$\frac{2F_{n-1}F_{n}+1}{F_{n}^{2}}\geq\frac{F_{n-1}F_{n+1}+1}{F_{n}^{2}}=\frac {F^{2}_{n}}{F_{n}^{2}}=1, $$
which implies that
$$\sum_{k=n}^{2n}\frac{1}{F_{k}^{2}}> \frac{1}{F_{n-1}F_{n}+1}+ \frac{1}{(F_{n-1}F_{n}+1)(F_{n}F_{n+1}+1)}-\frac{1}{F_{2n}F_{2n+1}+1}. $$
By (2.3) and (2.4), we have
$$\begin{aligned} F_{2n}F_{2n+1}+1 >&(F_{n-1}F_{n}+F_{n}F_{n+1}) \bigl(F_{n}^{2}+F_{n+1}^{2}\bigr) \\ >&(F_{n-1}F_{n}+1) (F_{n}F_{n+1}+1), \end{aligned}$$
from which we conclude that
$$\sum_{k=n}^{2n}\frac{1}{F_{k}^{2}}> \frac{1}{F_{n-1}F_{n}+1}. $$
The proof is complete. □
Proposition 3.4
Suppose that \(m\geq2\) and \(n>0\) is even. Then
$$ \sum_{k=n}^{mn} \frac{1}{F_{k}^{2}}< \frac{1}{F_{n-1}F_{n}-1}. $$
(3.4)
Proof
Applying (2.1), we can rewrite \(F_{k}^{4}\) as
$$ F_{k}^{4}=F_{k-1}F_{k}^{2}F_{k+1}+(-1)^{k-1}F_{k}^{2}. $$
(3.5)
In addition,
$$\begin{aligned} (F_{k-1}F_{k}-1) (F_{k}F_{k+1}-1) =&F_{k-1}F_{k}^{2}F_{k+1}-F_{k-1}F_{k}-F_{k}F_{k+1}+1 \\ =&F_{k-1}F_{k}^{2}F_{k+1}-2F_{k-1}F_{k}-F_{k}^{2}+1. \end{aligned}$$
(3.6)
Combining (3.5) and (3.6) yields
$$\begin{aligned} \frac{1}{F_{k-1}F_{k}-1}-\frac{1}{F_{k}^{2}}-\frac{1}{F_{k}F_{k+1}-1} =& \frac{F_{k}^{2}}{(F_{k-1}F_{k}-1)(F_{k}F_{k+1}-1)}- \frac{1}{F_{k}^{2}} \\ =&\frac {2F_{k-1}F_{k}-1+F_{k}^{2}+(-1)^{k-1}F_{k}^{2}}{(F_{k-1}F_{k}-1)(F_{k}F_{k+1}-1)F_{k}^{2}} \\ \geq&\frac{2F_{k-1}F_{k}-1}{(F_{k-1}F_{k}-1)(F_{k}F_{k+1}-1)F_{k}^{2}} \\ >&0. \end{aligned}$$
Therefore,
$$\sum_{k=n}^{mn}\frac{1}{F_{k}^{2}}< \frac{1}{F_{n-1}F_{n}-1}- \frac{1}{F_{mn}F_{mn+1}-1}<\frac{1}{F_{n-1}F_{n}-1}, $$
which completes the proof. □
Proposition 3.5
If \(n>0\) is even, then
$$ \sum_{k=n}^{2n} \frac{1}{F_{k}^{2}}> \frac{1}{F_{n-1}F_{n}}. $$
(3.7)
Proof
Employing (2.1), we can deduce that
$$ \frac{1}{F_{k-1}F_{k}}-\frac{1}{F_{k}^{2}}-\frac{1}{F_{k}F_{k+1}}= \frac{(-1)^{k-1}}{F_{k-1}F_{k}^{2}F_{k+1}}. $$
Hence, since n is even, we have
$$\begin{aligned} \begin{aligned} \sum_{k=n}^{2n}\frac{1}{F_{k}^{2}}={}& \frac{1}{F_{n-1}F_{n}}+ \sum_{k=n}^{2n} \frac{(-1)^{k}}{F_{k-1}F_{k}^{2}F_{k+1}}-\frac {1}{F_{2n}F_{2n+1}} \\ ={}&\frac{1}{F_{n-1}F_{n}}+ \biggl(\frac{1}{{F_{n-1}F_{n}^{2}F_{n+1}}}-\frac {1}{F_{n}F_{n+1}^{2}F_{n+2}} - \frac{1}{F_{2n}F_{2n+1}} \biggr) \\ &{}+\sum_{k=n+2}^{2n-1}\frac{(-1)^{k}}{F_{k-1}F_{k}^{2}F_{k+1}}+ \frac{1}{F_{2n-1}F_{2n}^{2}F_{2n+1}}. \end{aligned} \end{aligned}$$
It is easy to see that
$$\sum_{k=n+2}^{2n-1}\frac{(-1)^{k-1}}{F_{k-1}F_{k}^{2}F_{k+1}}>0, $$
thus
$$\sum_{k=n}^{2n}\frac{1}{F_{k}^{2}}> \frac{1}{F_{n-1}F_{n}}+ \biggl( \frac{1}{F_{n-1}F_{n}^{2}F_{n+1}}-\frac{1}{F_{n}F_{n+1}^{2}F_{n+2}} - \frac{1}{F_{2n}F_{2n+1}} \biggr). $$
We claim that
$$ \frac{1}{F_{n-1}F_{n}^{2}F_{n+1}}-\frac{1}{F_{n}F_{n+1}^{2}F_{n+2}} -\frac{1}{F_{2n}F_{2n+1}}>0. $$
First, by (2.6), we have
$$ \frac{1}{F_{n-1}F_{n}^{2}F_{n+1}}-\frac{1}{F_{n}F_{n+1}^{2}F_{n+2}}-\frac {1}{F_{2n}F_{2n+1}}= \frac{F_{2n+1}}{F_{n-1}F_{n}^{2}F_{n+1}^{2}F_{n+2}}- \frac{F_{2n+1}}{F_{2n}F_{2n+1}^{2}}. $$
It follows from (2.3), (2.4), and (2.5) that
$$\begin{aligned}& F_{2n} > F_{n-1}F_{n}, \\& F_{2n+1} > F_{n+1}^{2}, \\& F_{2n+1} > F_{n}F_{n+2}, \end{aligned}$$
which implies that
$$F_{2n}F_{2n+1}^{2}>F_{n-1}F_{n}^{2}F_{n+1}^{2}F_{n+2}. $$
Thus we obtain
$$\frac{1}{F_{n-1}F_{n}^{2}F_{n+1}}-\frac{1}{F_{n}F_{n+1}^{2}F_{n+2}} -\frac{1}{F_{2n}F_{2n+1}}>0, $$
which yields the desired (3.7). □
Now we introduce our main result on the square sum of reciprocal Fibonacci numbers.
Theorem 3.6
For all \(n\geq1\) and \(m\geq2\), we have
$$ \Biggl\lfloor \Biggl(\sum_{k=n}^{mn} \frac{1}{F_{k}^{2}} \Biggr)^{-1} \Biggr\rfloor = \left \{ \begin{array}{l@{\quad}l} F_{n}F_{n-1}, &\textit{if }n\textit{ is odd}; \\ F_{n}F_{n-1}-1, &\textit{if }n\textit{ is even}. \end{array} \right . $$
(3.8)
Proof
We first consider the case when n is odd. If \(n=1\), the result is clearly true. So we assume that \(n\geq3\).
It follows from (3.3) that
$$ \sum_{k=n}^{mn} \frac{1}{F_{k}^{2}}\geq\sum_{k=n}^{2n} \frac{1}{F_{k}^{2}}> \frac{1}{F_{n-1}F_{n}+1}. $$
(3.9)
Employing (3.2) and (3.9) yields
$$\frac{1}{F_{n-1}F_{n}+1}< \sum_{k=n}^{mn} \frac{1}{F_{k}^{2}}<\frac{1}{F_{n-1}F_{n}}, $$
which implies that, if \(n>0\) is odd, we have
$$\Biggl\lfloor \Biggl(\sum_{k=n}^{mn} \frac{1}{F_{k}^{2}} \Biggr)^{-1} \Biggr\rfloor =F_{n}F_{n-1}. $$
We now consider the case where \(n>0\) is even. It follows from (3.7) that
$$ \sum_{k=n}^{mn} \frac{1}{F_{k}^{2}}\geq\sum_{k=n}^{2n} \frac{1}{F_{k}^{2}}> \frac{1}{F_{n-1}F_{n}}. $$
(3.10)
Combining (3.4) and (3.10), we arrive at
$$\frac{1}{F_{n-1}F_{n}}< \sum_{k=n}^{mn} \frac{1}{F_{k}^{2}}<\frac{1}{F_{n-1}F_{n}-1}, $$
from which we find that, if \(n>0\) is even,
$$\Biggl\lfloor \Biggl(\sum_{k=n}^{mn} \frac{1}{F_{k}^{2}} \Biggr)^{-1} \Biggr\rfloor =F_{n}F_{n-1}-1. $$
This completes the proof. □
Remark
Theorem 1.2 can be regarded as the limiting case as \(m\rightarrow\infty\) in (3.8).

Acknowledgements

This work was supported by the National Natural Science Foundation of China (No. 11401080) and the Research Project of Education Teaching Reform of University of Electronic Science and Technology of China (No. 2013XJYEL032). The authors would like to thank the referees for helpful comments leading to an improvement of an earlier version.
Open Access This is an Open Access article distributed under the terms of the Creative Commons Attribution License (http://creativecommons.org/licenses/by/4.0), which permits unrestricted use, distribution, and reproduction in any medium, provided the original work is properly credited.

Competing interests

The authors declare that they have no competing interests.

Authors’ contributions

All authors contributed equally to deriving all the results of this article, and read and approved the final manuscript.
Literatur
2.
Zurück zum Zitat Duncan, RL: Applications of uniform distribution to the Fibonacci numbers. Fibonacci Q. 5, 137-140 (1967) MATH Duncan, RL: Applications of uniform distribution to the Fibonacci numbers. Fibonacci Q. 5, 137-140 (1967) MATH
4.
Zurück zum Zitat Ma, R, Zhang, WP: Several identities involving the Fibonacci numbers and Lucas numbers. Fibonacci Q. 45, 164-170 (2007) MATH Ma, R, Zhang, WP: Several identities involving the Fibonacci numbers and Lucas numbers. Fibonacci Q. 45, 164-170 (2007) MATH
6.
Zurück zum Zitat Elsner, C, Shimomura, S, Shiokawa, I: Algebraic relations for reciprocal sums of Fibonacci numbers. Acta Arith. 130, 37-60 (2007) CrossRefMATHMathSciNet Elsner, C, Shimomura, S, Shiokawa, I: Algebraic relations for reciprocal sums of Fibonacci numbers. Acta Arith. 130, 37-60 (2007) CrossRefMATHMathSciNet
7.
Zurück zum Zitat Elsner, C, Shimomura, S, Shiokawa, I: Algebraic relations for reciprocal sums of odd terms in Fibonacci numbers. Ramanujan J. 17, 429-446 (2008) CrossRefMATHMathSciNet Elsner, C, Shimomura, S, Shiokawa, I: Algebraic relations for reciprocal sums of odd terms in Fibonacci numbers. Ramanujan J. 17, 429-446 (2008) CrossRefMATHMathSciNet
8.
Zurück zum Zitat Elsner, C, Shimomura, S, Shiokawa, I: Algebraic independence results for reciprocal sums of Fibonacci numbers. Acta Arith. 148, 205-223 (2011) CrossRefMATHMathSciNet Elsner, C, Shimomura, S, Shiokawa, I: Algebraic independence results for reciprocal sums of Fibonacci numbers. Acta Arith. 148, 205-223 (2011) CrossRefMATHMathSciNet
9.
Zurück zum Zitat Elsner, C, Shimomura, S, Shiokawa, I: Algebraic relations for reciprocal sums of even terms in Fibonacci numbers. J. Math. Sci. 180, 650-671 (2012) MATHMathSciNet Elsner, C, Shimomura, S, Shiokawa, I: Algebraic relations for reciprocal sums of even terms in Fibonacci numbers. J. Math. Sci. 180, 650-671 (2012) MATHMathSciNet
10.
Zurück zum Zitat Ohtsuka, H, Nakamura, S: On the sum of reciprocal Fibonacci numbers. Fibonacci Q. 46/47, 153-159 (2008/2009) Ohtsuka, H, Nakamura, S: On the sum of reciprocal Fibonacci numbers. Fibonacci Q. 46/47, 153-159 (2008/2009)
11.
Zurück zum Zitat Wu, ZG, Zhang, WP: The sums of the reciprocals of Fibonacci polynomials and Lucas polynomials. J. Inequal. Appl. 2012, Article ID 134 (2012) CrossRef Wu, ZG, Zhang, WP: The sums of the reciprocals of Fibonacci polynomials and Lucas polynomials. J. Inequal. Appl. 2012, Article ID 134 (2012) CrossRef
12.
Zurück zum Zitat Wu, ZG, Zhang, WP: Several identities involving the Fibonacci polynomials and Lucas polynomials. J. Inequal. Appl. 2013, Article ID 205 (2013) CrossRef Wu, ZG, Zhang, WP: Several identities involving the Fibonacci polynomials and Lucas polynomials. J. Inequal. Appl. 2013, Article ID 205 (2013) CrossRef
13.
Zurück zum Zitat Holliday, SH, Komatsu, T: On the sum of reciprocal generalized Fibonacci numbers. Integers 11A, Article ID 11 (2011) MathSciNet Holliday, SH, Komatsu, T: On the sum of reciprocal generalized Fibonacci numbers. Integers 11A, Article ID 11 (2011) MathSciNet
14.
Zurück zum Zitat Wu, ZG, Wang, TT: The finite sum of reciprocal of the Fibonacci numbers. J. Inn. Mong. Norm. Univ. Nat. Sci. 40, 126-129 (2011) Wu, ZG, Wang, TT: The finite sum of reciprocal of the Fibonacci numbers. J. Inn. Mong. Norm. Univ. Nat. Sci. 40, 126-129 (2011)
Metadaten
Titel
On the partial finite sums of the reciprocals of the Fibonacci numbers
verfasst von
Andrew YZ Wang
Peibo Wen
Publikationsdatum
01.12.2015
Verlag
Springer International Publishing
Erschienen in
Journal of Inequalities and Applications / Ausgabe 1/2015
Elektronische ISSN: 1029-242X
DOI
https://doi.org/10.1186/s13660-015-0595-6

Weitere Artikel der Ausgabe 1/2015

Journal of Inequalities and Applications 1/2015 Zur Ausgabe

Premium Partner